AIME 2015 Problem 10

Call a permutation a 1 , a 2 , , a n a_1, a_2, \ldots, a_n of the integers 1 , 2 , , n 1, 2, \ldots, n quasi-increasing if a k a k + 1 + 2 a_k \leq a_{k+1} + 2 for each 1 k n 1 1 \leq k \leq n-1 . For example, 53421 53421 and 14253 14253 are quasi-increasing permutations of the integers 1 , 2 , 3 , 4 , 5 1, 2, 3, 4, 5 , but 45123 45123 is not. Find the number of quasi-increasing permutations of the integers 1 , 2 , , 7 1, 2, \ldots, 7 .


Try more problems of AIME from this set AIME 2015


The answer is 486.

This section requires Javascript.
You are seeing this because something didn't load right. We suggest you, (a) try refreshing the page, (b) enabling javascript if it is disabled on your browser and, finally, (c) loading the non-javascript version of this page . We're sorry about the hassle.

1 solution

Brandon Monsen
Dec 5, 2015

We can find a recursive formula for the number of quasi-increasing sequences of the first n n natural numbers.

Let the number of quasi-increasing sequences for the first n n natural numbers be s n s_{n} .

s 1 = 1 s_{1}=1 and for n = 2 n=2 , the 2 2 can go anywhere, so s 2 = 2 ! s_{2}=2! . For n = 3 n=3 , you can put the 3 3 anywhere on any sequence for n = 2 n=2 , so s 3 = 3 ! s_{3}=3! .

Now, for n = 4 n=4 , you can't put the 4 4 before the 1 1 , so there are 4 1 4-1 more options for where to put the 4 4 . This means s 4 = 3 ! × 3 s_{4}=3! \times 3 .

For n = 5 n=5 , you can't put the 5 5 before the 2 2 or 1 1 , so there are 5 2 5-2 places to put the 5 5 , thus s 5 = 3 ! × 3 2 s_{5}=3! \times 3^{2} .

This pattern will hold for s 6 s_{6} and s 7 s_{7} , so we know that s 7 = 3 ! × 3 4 = 486 s_{7}=3! \times 3^{4}=\boxed{486}

same method, glad I didn't bash it

keanu ac - 4 years ago

0 pending reports

×

Problem Loading...

Note Loading...

Set Loading...